Kommutierung des Hamiltonoperators mit Impuls

In diesem Fall gilt der Hamiltonoperator H pendelt mit dem Impuls P ?

Kann mir jemand helfen? Mit einem Beispiel? (Keine besonderen oder seltsamen Hamiltonianer und keine besonderen Impulse sind beteiligt).

Wie kann ich das beweisen [ H , P ] = 0 ?

Vielleicht nur dann, wenn ich eine freie Theorie ohne Potenzial habe?
@KyleKanos Ich habe gerade die Berechnung durchgeführt und erhalten [H, P] = -ih d / dx v (x) Das ist Null, wenn V = 0, das ist in der freien Theorie!
Welche Art von Antwort suchen Sie? Einer könnte das sein [ H , P ] = 0 wenn das System translationsinvariant ist
@glance gilt meine obige Berechnung?
@Henry welche Berechnung? Die Tatsache, dass [ P 2 , P ] = 0 ? Nun ja, wenn das Ihre Frage war. Das Vorhandensein eines Potentials unterbricht die Translationsinvarianz, wodurch der Hamilton-Operator nicht mit dem Translationsgenerator pendelt, dh P
@glance nein, ich meinte meine Antwort! Die Berechnung war trivial, und ich erhielt -ih d/dx V(x). Darf ich denken, dass [H, P] = 0 für Nullpotential, und ich kann darüber als freie Theorie (oder Vakuumtheorie?)
@glance Translationsinvarianz, was bedeutet, dass V wie aussehen sollte?
@Sofia Ich verstehe deine Frage nicht. Das bedeutet das Hamiltonsche Pendeln mit dem Übersetzungsgenerator (in 1D). X , A , v ( X + A ) = v ( X ) , also ist das Potential konstant..

Antworten (3)

Wenn der Hamiltonoperator unter Translationen invariant ist. Um dies zu sehen, erinnere dich daran P ist der infinitesimale Generator von Übersetzungen. Wie zB Dirac in Lectures on Quantum Mechanics gezeigt hat, kommutiert jeder infinitesimale Generator einer Symmetrie mit dem Hamiltonoperator, der selbst der Generator der Zeittranslationen, dh der Dynamik ist.

Typische Beispiele für einen Hamiltonian, der mit pendelt P das freie Teilchen ist, oder allgemeiner jede zulässige Funktion von P allein. Das QHO ist ein Beispiel, wo eine solche Kommutierung nicht gilt, da das harmonische Potential die Symmetrie unter Translation (und natürlich eine Funktion der Positionen) deutlich bricht Q möglicherweise nicht mit pendeln P ).

Kann ich bitte die Berechnung sehen? Ich meine die Kommutierungsrelation der infinitesimalen Generatoren, die mit H kommutieren. Auch die Buchseite wäre toll!
es kommt nur von der Erweiterung e ich P A H ( X ) e ich P A = H ( X ) um A = 0 und Gleichsetzung von Begriffen bei jeder Bestellung. Die gesuchte Beziehung stammt aus dem IIRC erster Ordnung

Hier ist ein schneller Beweis:

[ H ^ , P ^ ] = [ T ^ + v , P ^ ] = [ P ^ 2 2 M + v , P ^ ] = [ P ^ 2 2 M , P ^ ] + [ v , P ^ ]
Beachten Sie, dass das Potential hier im Allgemeinen eine Funktion von x ist, dh v ( X ) . Nutzung der Eigenschaft von Kommutatoren, die:
[ A B , C ] = A [ B , C ] + [ A , C ] B

Verwenden Sie auch das Ergebnis that für eine beliebige Funktion F :

[ F , P ^ ] = ich F X

Wir bekommen:

[ H ^ , P ^ ] = 1 2 M ( P ^ [ P ^ , P ^ ] + [ P ^ , P ^ ] P ^ ) + ich v X

Operator pendelt mit sich selbst! So [ P ^ , P ^ ] = 0 :

[ H ^ , P ^ ] = ich v X

Wenn v X = 0 , dh v hat keine explizite Abhängigkeit von X , Dann:

[ H ^ , P ^ ] = 0

Ist nicht P 2 der Skalaroperator P P ? In diesem Fall rechnen Sie tatsächlich [ A B , C ] … ist es das Gleiche?
@ric.san: ja, du hast recht. Genau das habe ich getan, um die Kommutatorbeziehung aufzuschreiben.

Der Hamilton-Operator für ein quantenmechanisches System wird durch die imaginäre Einheit multipliziert mit der partiellen Zeitableitung dargestellt. Der Impuls ist proportional zum Gradienten. Wenn Sie ein System in Bezug auf zwei unabhängige Variablen ableiten (was die partielle Ableitung tut, sie ignoriert Ihre Position als Funktion der Zeit), spielt es keine Rolle, auf welche Sie es zuerst ableiten.

Daher die zeitliche Ableitung und der Gradientenpendel.

Da die Proportionalitätskoeffizienten konstante Skalare sind, tauschen sie auch mit den beiden Ableitungen aus, wodurch sich alles aufhebt und Null ergibt. Ich weiß nicht, wie man hier Gleichungen aufstellt, also ist dies das Beste, was ich Ihnen geben kann, es sei denn, dies funktioniert:

[ P J , H ] ψ = ich X J ich T ψ ich T ( ich ) X J ψ = 2 ( X J T T X J ) ψ = 2 [ X J , T ] ψ
Aber T Und X J (Die J th räumliche Variable wo X 1 ist der X -Koordinate, X 2 = j ,   X 3 = z ) werden unabhängig von den partiellen Ableitungen behandelt (wenn sie vollständig abgeleitet wurden D D T dann würden Sie einige räumliche Koordinaten in Geschwindigkeit umwandeln und so), was das bedeutet [ X J , T ] = 0 . Somit [ P J , H ] = 0 Und [ P , H ] = 0 .

Diese Antwort ist falsch, der Hamiltonoperator ist nicht der Zeitableitungsoperator (der selbst kein Operator im Hilbert-Raum ist) und alles Folgende ist folglich ebenfalls falsch. Die Antwort von Phoenix87 ist gut.
Um den vorherigen Kommentar zu erweitern: Der Hamilton-Operator wird durch eine geeignete Kombination von Operatoren dargestellt, die Beiträge zur Energie darstellen. Das stimmt nicht H ^ = ich / T . Vielmehr ist die physische Entwicklung im Laufe der Zeit so, dass Kets die Schr befriedigen. Gleichung. Die Tatsache, dass zwei Operatoren die gleiche Wirkung auf eine Teilmenge aller Kets haben, bedeutet nicht, dass die Operatoren gleich sind, obwohl sie in diesem Beispiel die gleiche Wirkung auf alle physikalisch zulässigen Kets haben.